A Small Restriction With A Huge Impact

Algebra Level 4

Positive integers a , b a, b and c c satisfy the condition that the quadratic equation a x 2 b x + c = 0 ax^2 - bx +c =0 has 2 distinct real roots that are strictly between 0 and 1. What is the minimum value of a a ?


The answer is 5.

This section requires Javascript.
You are seeing this because something didn't load right. We suggest you, (a) try refreshing the page, (b) enabling javascript if it is disabled on your browser and, finally, (c) loading the non-javascript version of this page . We're sorry about the hassle.

4 solutions

Pranshu Gaba
Apr 5, 2014

Let f ( x ) = a x 2 b x + c f(x) = ax^2 - bx +c

Using properties of quadratic equations we get many inequalities. The following ones will be useful.

b 2 > 4 a c b^2 > 4ac a a a a a \phantom{aaaaa} ( f ( x ) = 0 f(x) = 0 has 2 2 distinct roots)

c a < 1 \frac{c}{a}< 1 a a a a a a a \phantom{aaaaaaa} (Product of roots of f ( x ) f(x) is between 0 0 and 1 1 )

a b + c > 0 a - b + c > 0 a a \phantom{aa} ( f ( 1 ) > 0 f(1) > 0 )

b a < 2 \frac{b}{a} < 2 a a a a a a a \phantom{aaaaaaa} (Sum of roots of f ( x ) f(x) is between 0 0 and 2 2 )

In addition, it is given that a , b , c > 0 a, b, c > 0


Rearranging the above inequalites gives

b 2 > 4 a c . . . . . . . . . . ( 1 ) b^2 >4ac .......... (1)

c < a . . . . . . . . . . . . . . ( 2 ) c < a ..............(2)

b < a + c . . . . . . . . . . . ( 3 ) b < a + c ...........(3)

b < 2 a . . . . . . . . . . . . . . ( 4 ) b < 2a ..............(4)

Since ( 2 ) (2) is true, ( 3 ) (3) implies ( 4 ) (4) , therefore ( 4 ) (4) becomes redundant.

Using the three inequalities, to minimize a a , by inspection we can see c c should be minimized, therefore c = 1 c = 1


Now, inequalities ( 1 ) (1) and ( 3 ) (3) become

b 2 > 4 a b^2 > 4a

b < a + 1 b < a + 1

The maximum value of b b for any value of a a can be obtained by setting b = a + 1 1 = a b = a + 1 - 1 = a

This gives us the final inequality a 2 > 4 a , a i.e. a > 4 a^2 > 4a, \phantom{a}\text{i.e. } a >4 .

We can see clearly, that a = 5 \boxed{a = 5} is the minimum integer value that satisfies the inequality.

This problem is so hard :(

Pil Pinas - 4 years, 11 months ago

Good solution.I too did in that way.Beautiful presentation.Upvoted!!

rajdeep brahma - 4 years, 2 months ago

Maximum value of a can be obtained by setting b = a + 1 1 b=a+1-1 .. How ?

Vishal Yadav - 4 years, 2 months ago

Log in to reply

Since b < a + 1 b < a + 1 , we can find b b by setting it as one integer less than a + 1 a + 1 which is a a

Pranshu Gaba - 4 years, 1 month ago

This reasoning is faulty: "Using the three inequalities, to minimize a, by inspection we can see c should be minimized, therefore c = 1." You said: a>c, so to obtain smallest a, let's put c=1. But, a is not 2, so c need not to be 1.

lovro cupic - 1 year, 9 months ago
Eloy Machado
Apr 8, 2014

Small a a results in big opening (more spread) parabola and big a a results in small opening.

So, if we want the minimum a a we should have the roots as far as possible to each one. It will be possible with axis of symmetry passing by x = 1 2 x = \frac{1}{2} .

It means ( b ) 2 a = 1 2 b = a \frac { -(-b) }{ 2a } =\frac { 1 }{ 2 } \therefore b=a

Because 2 distinct roots, b 2 4 a c > 0 b 2 > 4 a c { b }^{ 2 }-4ac>0\Rightarrow { b }^{ 2 }>4ac

plugging b = a b=a in last inequality, we get: a 2 > 4 a c a > 4 c { a }^{ 2 }>4ac \Rightarrow a > 4c

So, to minimize a a we need minimize c c . The minimum (positive and integer) c c is c = 1 c = 1 , so we have the minimum a a is a = 5 \boxed{a = 5} .

and we are done.

wow beauty!!

Abhinav Raichur - 6 years, 1 month ago

excellent view !!

space sizzlers - 4 years, 11 months ago

Log in to reply

Reaally good way

Ravendra Kumar - 4 years, 10 months ago

The first step is the killer step.Mindblowing approach.I learned a lot from it.Upvoted!!

rajdeep brahma - 4 years, 2 months ago

Really nice and elegant! I started up with exactly the same geometrical analysis but took more steps to the final solution.

Gabriel Chacón - 2 years, 4 months ago

This reasoning is faulty: "It will be possible with axis of symmetry passing by x = 1 2 x = \frac{1}{2} ." If the interval was different, say 0 , 2 , b a \langle 0, \sqrt{2} \rangle\:, \quad \frac{b}{a} would not be rational if axis of symmetry passed through the middle of the interval. In other words, to claim that any parabola satisfying given conditions can be centered that way, one must be certain that translation necessarily leaves coefficients in N \mathbb{N} , which is not the case. Translation of y = a x 2 + b x + c for t (vector ( t , 0 ) ) acts on b , c so that y t = a x 2 + ( b 2 a t ) x + c b t + a t 2 y=ax^2+bx+c\text{ for } t \text{ (vector } (t,0)\:) \text{ acts on } b,c \text{ so that } y_t=ax^2+(b-2at)x+c-bt+at^2

lovro cupic - 1 year, 9 months ago

Log in to reply

Hmm looks like your reasoning is correct. But I suppose this one works fine for naturals . But hey we can cheat a little bit by approximating like as you pointed about √2 one we can "approximate" by saying that √2 is just so about equal to 1.5 Then taking 0.75 as vertex our work can be done. By the way I suppose the answer for interval of √2 is 2

Hitesh Yadav - 11 months, 2 weeks ago
Pi Han Goh
Apr 4, 2014

By Vieta's Formula, the sum of roots is b a \frac {b}{a} and the product of roots is c a \frac {c}{a} . Because the distinct real roots are between 0 0 and 1 1 exclusive. c a < 1 c < a \frac {c}{a} < 1 \Rightarrow c < a and b a < 1 + 1 b < 2 a \frac {b}{a} < 1+1 \Rightarrow b < 2a . Noting that 6 ( x 1 2 ) ( x 1 3 ) = 6 x 2 5 x + 1 6 \left (x - \frac {1}{2} \right ) \left ( x - \frac {1}{3} \right ) = 6x^2 - 5x + 1 satisify the condition, we just need to check for a 6 a \leq 6

  • a = 1 a = 1 : there's no solution for c c

  • a = 2 a = 2 : c = 1 c = 1 only, b = 1 , 2 , 3 b = 1,2,3 , because they are real roots, the determinant must be positive, b 2 4 a c > 0 b = 3 b^2 - 4ac > 0 \Rightarrow b = 3 only. Solve 2 x 2 3 x + 1 = 0 2x^2-3x+1 =0 gives x = 1 x=1 as a solution, which is invalid.

  • a = 3 a = 3 : apply the same logic above for the determinant, we have ( b , c ) = ( 4 , 1 ) , ( 5 , 1 ) , ( 5 , 2 ) (b,c) = (4,1), (5,1), (5,2) . Trial and error shows none of these satisfy the condition.

  • a = 4 a=4 : ( b , c ) = ( 5 , 1 ) , ( 6 , 1 ) , ( 6 , 2 ) , ( 7 , 1 ) , ( 7 , 2 ) , ( 7 , 3 ) (b,c) = (5,1),(6,1),(6,2),(7,1),(7,2),(7,3) . No solution.

  • a = 5 a = 5 : ( b , c ) = ( 5 , 1 ) , (b,c) = (5,1), \ldots . Turns out the very first pair works out, so ( b , c ) = ( 5 , 1 ) (b,c) = (5,1) is valid. The answer is 5 \boxed{5}

P.S: Once we conclude that a = 1 , 2 , 3 a=1,2,3 don't work. And we know that at least one of a = 4 , 5 , 6 a=4,5,6 must work. Plus, we are given 3 tries....

Yeah...I too got 1,2,3 are not favourable.... Then between 4,5 I guessed 5 first!!! Lucky me :)

Tanya Gupta - 7 years, 2 months ago

Log in to reply

I also didn't check for 4 and 5, luckily I guessed 5

Mardokay Mosazghi - 7 years, 2 months ago
Marco Antonio
Jan 5, 2016

Here it is my way to resolve it!

a x 2 b x + c = 0 x = b ± b 2 4 a c 2 a ax^{2} - bx + c = 0 \Longrightarrow x = \frac{b \pm \sqrt{b^{2} - 4ac} }{2a} Therefore we need to define two things: the roots of the equation must be real; the roots must be less than 1 1 and greater than 0 0 . Looking for the equation, we can do that:

For the real roots:
b 2 4 a c > 0 a < b 2 4 c b^{2} - 4ac > 0 \Longrightarrow a < \frac{b^{2}}{4c} ;

For 0 < x < 1 0<x<1 :
2 a > b ± b 2 4 a c a > b c 2a > b \pm \sqrt{b^{2} - 4ac} \Longrightarrow a > b - c ;

We get this:
b c < a < b 2 4 c b-c < a < \frac{b^{2}}{4c} .

Now, we can play with this inequality until we get 5 5 .

And there's a lot of sweet information there like when c = b 2 c = \frac{b}{2} , b c b-c and b 2 4 c \frac{b^{2}}{4c} are equal (which I'd like to know how this happens from the "master" since it looks like too difficult for me understand and work the interception of two functions with two variables - just some titles of what I have to read and study it's enough). I almost failed for thinking that b 2 4 c \frac{b^{2}}{4c} also must be an integer. What a dumb conclusion.

Moderator note:

Can you add more details to "Now we can play with this inequality until we get 5"? That is the crux of this problem, which I feel isn't sufficiently explained.

Can you add more details to "Now we can play with this inequality until we get 5"? That is the crux of this problem, which I feel isn't sufficiently explained.

Calvin Lin Staff - 5 years, 5 months ago

Log in to reply

Well. I had to recall my messy paper. I did this way: First, look how this expression behaviors with some values. Then, I made an analysis: for c = b 2 c = \frac{b}{2} , I get b 2 < a < b 2 \frac{b}{2} < a < \frac{b}{2} , which tells me two things: first, this cannot be my solution, second: c b c-b and b 2 4 c \frac{b^2}{4c} has an interception point, which I don't know weather it's the minimum of b 2 4 c \frac{b^2}{4c} or not. So I try to plug other values bigger than b 2 \frac{b}{2} for c c in terms of b b and see what I got, and it turns out that b 2 4 c \frac{b^2}{4c} became bigger than c b c-b . For example (for c = 3 b 4 c = \frac{3b}{4} ): b 3 b 4 < a < b 2 4 × 3 b 4 b 4 < a < b 3 b - \frac{3b}{4} < a < \frac{b^2}{\frac{4 \times 3b}{4}} \Rightarrow \frac{b}{4} < a < \frac{b}{3} . Something like: these two expressions has a period with the same images but after that it becomes to be different again (the left part smaller than the right part) . I hope I expressed myself well till here.

I had in mind that I was looking for the minimum value of a a , and a a , b b and c c must be positive integers. The second thing I noticed is that, I have to watch out for my expression because she's telling me that almost every function can have a a = 1 a = 1 and still have roots between 0 < x < 1 0 < x < 1 . I wasn't sure why but I had some clues that it's related to the sum and product of the roots by Vieta that can gives me an approximation like c < a c < a . Since I didn't even "know" about Vieta formulas, I was even considering the case of c < b c < b - but with the comments, now I know :D -. With the last piece of cake, I realize that b 2 4 c \frac{b^2}{4c} doesn't have to be integer, and this was driving me crazy!!!

How all those information helped me? I knew what values of c c and b b I could plug in and what I should expect of it. And the first pair of numbers was a = 5 , b = 5 , c = 1 a = 5, b = 5, c = 1 . For c > 1 c > 1 it wouldn't work and for b > 5 b > 5 the answer would grow up, I gave it my last try. And it works.

Looking now for my answer, it maybe looks like a lit of bit of a messy.

Marco Antonio - 5 years, 5 months ago

Log in to reply

Ah yes. That approach will require quite a lot of trial and error, like you stated.

Calvin Lin Staff - 5 years, 5 months ago

0 pending reports

×

Problem Loading...

Note Loading...

Set Loading...